LSAT and Law School Admissions Forum

Get expert LSAT preparation and law school admissions advice from PowerScore Test Preparation.

User avatar
 Dave Killoran
PowerScore Staff
  • PowerScore Staff
  • Posts: 5853
  • Joined: Mar 25, 2011
|
#41323
Setup and Rule Diagram Explanation

This is a Basic Linear: Unbalanced: Overloaded game.

Because we have eight apartments that must be distributed over five floors, this is an Unbalanced:
Overloaded game. The given rules and restrictions can then be used to create a 2-2-2-1-1 unfixed numerical distribution. Unfortunately, this distribution proves to be of little value in the game. This occurrence should not deter you from seeking numerical distributions in the future. There are many examples of games where the numerical distribution proved to be the key to answering one or more questions.

The game should be set up as follows:

pt2_o91_g2_1.png

One difficult aspect of this game is the uncertainty over which floors contain two apartments. Since it is certain that there is at least one apartment per floor, we have placed slots on each of the five floors of our diagram. In the case of the second floor, which is known to have only one apartment, a short vertical line has been placed at the end of the slot. This vertical line serves as a visual reminder that the second floor contains one and only one apartment. Also of note in this game is the importance of correctly diagramming each rule. In the case of K, who lives one floor above P, the block must be shown vertically since the main diagram is vertical. On the other hand, the rule involving M and N must be shown horizontally because they live on the same floor. By diagramming these rules correctly, you gain a powerful advantage over the game, and you also eliminate a possible source of confusion. Also note that if this game were set up horizontally, then the diagramming of each rule would shift accordingly. For example, the KP block would be horizontal, whereas the MN block would be vertical. In essence you align the blocks with the diagram in order to make the most visual sense. On a vertical diagram a vertical block suggests one variable on top of another, but on a horizontal diagram a vertical block suggests that the two variables share the same space.

Let us take a moment to examine the Not Laws in the game:

The occupancy limitation on the second floor produces Not Laws for J, M, and N, each of which is involved in a block rule.

Because K must live on the floor directly above P, P cannot live on the fifth floor and K cannot live on the first floor.

The last rule creates Q Not Laws on the first two floors. Also note that although Q cannot live on the first and second floors, this does not affect the placement of O. In Linear games, not-blocks tend to be relatively weak rules (this will not be the case when discussing grouping games), because the not-block cannot be applied until one of the variables in the block is placed. Since Q has not been placed, it has not yet had an effect on O.
One of the keys to doing well on the questions is to remember all of the different rules, each of which is unique in form (this is, of course, why we represent rules visually—doing so makes them easier to remember).
You do not have the required permissions to view the files attached to this post.
 lakasil
  • Posts: 10
  • Joined: Aug 08, 2016
|
#28713
On page 188-190 Why does " J,M,N," not belong on floor 2 and how does it reference to the rules of this game?
 Nikki Siclunov
PowerScore Staff
  • PowerScore Staff
  • Posts: 1362
  • Joined: Aug 02, 2011
|
#28846
Hi lakasil,

Thanks for your question. As explained on page 191, the occupancy of floor 2 is limited to only one apartment. Since M and N must be on the same floor as each other, neither of them can be on floor 2. Likewise, since J lives of a floor with two apartments, she cannot live on the second floor.

Hope this helps! :)
 btownsquee
  • Posts: 13
  • Joined: Mar 01, 2017
|
#33999
Why is the numerical distribution 2-2-2-1-1 in this game? Why is 3-2-1-1-1 not a possibility?

Thanks!
User avatar
 Dave Killoran
PowerScore Staff
  • PowerScore Staff
  • Posts: 5853
  • Joined: Mar 25, 2011
|
#34048
Hey btown,

Thanks for the question! You can't have 3 apartments on a floor because of the second sentence of the game scenario: "Each floor has either one or two apartments."

This is another one of those situations where important rule-type info is included in the scenario, and reinforces the importance of making sure you closely examine the scenario each time. While most of them just contain basic "cleanup" language, occasionally they add rules in there as well!

Thanks!
 hope
  • Posts: 84
  • Joined: Feb 13, 2018
|
#80253
WHY CAN'T Q BE ON THE FIRST FLOOR AND FOR THAT MATTER WHY CAN'T Q BE ON THE SECOND FLOOR ALL BY ITSELF? I DON'T READ THE OQ NOT BLOCK AS STATING THAT Q CAN'T BE ON THE 1ST OR 2ND FLOOR. CAN SOMEONE RESPOND AT YOUR EARLIEST CONVENIENCE? THANK YOU. :-D
 Paul Marsh
PowerScore Staff
  • PowerScore Staff
  • Posts: 290
  • Joined: Oct 15, 2019
|
#80356
Hi Hope!

The last rule of the game directly states, "Q does not live on the first or second floor." So that's why we can't have Q on either of those floors.

Hope that helps!

Get the most out of your LSAT Prep Plus subscription.

Analyze and track your performance with our Testing and Analytics Package.